LSAT and Law School Admissions Forum

Get expert LSAT preparation and law school admissions advice from PowerScore Test Preparation.

User avatar
 Dave Killoran
PowerScore Staff
  • PowerScore Staff
  • Posts: 5853
  • Joined: Mar 25, 2011
|
#44236
Complete Question Explanation
(The complete setup for this game can be found here: lsat/viewtopic.php?t=16650)

The correct answer choice is (D)

According to the question stem, K and H give afternoon reports on Tuesday and Wednesday, respectively. This leaves Monday afternoon as the most restricted space: O and R cannot give reports on Monday afternoon, G cannot give a report on Monday, and N cannot give a report on Monday since K gives a report on Tuesday. Thus, either I or L must give a report on Monday afternoon:
O00_Game_#1_#5_diagram 1.png
Each of the four incorrect answers contains both I and L as students giving morning reports. Answer choice (D) is thus correct.
You do not have the required permissions to view the files attached to this post.
 glenm
  • Posts: 4
  • Joined: Jan 11, 2013
|
#7391
Lesson #7 hw pg 7-98
I don't know what I am missing I have read the explanation in the book, I don't understand why R cant be in the morning on question #5
 Steve Stein
PowerScore Staff
  • PowerScore Staff
  • Posts: 1153
  • Joined: Apr 11, 2011
|
#7394
Hey Glen,

Thanks for your question. In that one, you are correct--Robert actually can be in the morning; the correct answer is D--Robert, George, and Irving, respectively.

Please let me know whether that clears this one up--thanks!

~Steve
 srcline@noctrl.edu
  • Posts: 243
  • Joined: Oct 16, 2015
|
#22005
Hello

With this game I didn't seem to have trouble with the set up by I am getting tripped up on the last couple of questions..... For example for question 5, would it be wrong to immediately cross out A, C, E, because George would have to give the Morning Monday report? Also we dont know if O or R is either Monday or Wednesday, so answer choice B is correct?

Thank you
Sarah
 Emily Haney-Caron
PowerScore Staff
  • PowerScore Staff
  • Posts: 577
  • Joined: Jan 12, 2012
|
#22013
Hi Sarah,

Remember that you have 8 students for 6 reports, and so it is possible that George does not give a report at all. With this set-up, we know that if N gives a report, it cannot be Monday (since HI can't go on Tuesday; H is on Wednesday).That eliminates answer C. After that, we're plugging in the variables given in each answer choice (which means we have every slot filled but Monday afternoon) and seeing whether one of the three variables left for Monday afternoon can go there without violating rules.

For A, the unused variables that could go on Monday afternoon are G, O, or R; but O and R can't go in the afternoon and G can only go on Tuesday. We can eliminate A.

For B, we are left with N, O, or R; O and R can't go in the afternoon, and We know that N can't go on Monday because HI can't go on Tuesday. We can eliminate B.

For D, we are left with G, L, and N. G and N both can't go on Monday. But there is no reason L can't; this is the correct answer.

For E, we are left with G, N, and O; O can't go in the afternoon, G and N cannot go on Monday. We can eliminate E.

Get the most out of your LSAT Prep Plus subscription.

Analyze and track your performance with our Testing and Analytics Package.